希望这里有一些计算几何学的人可以帮助我解决以下问题 -
请想象我在 3 空间中取一个自由移动的球,并通过定义一组不可通过的坐标 Sc(即 3 空间中不允许扩散球的任何部分重叠的点)在它周围创建一个“笼子”。这些点位于某个较大球体的体积 V(cage) 内,其中 V(cage) >> V(ball)。
假设一组不可通过的坐标 Sc,是否有一种计算上有效和/或很好的方法来确定球是否可以逃脱笼子?
请参阅我之前在 MathOverflow 上的帖子 - https://mathoverflow.net/questions/21911/when-can-a-freely-moving-sphere-escape-from-a-cage-defined-by-a-set-of-无言以对